Difference between revisions of "2007 AMC 12A Problems/Problem 15"

(See also)
m
 
Line 8: Line 8:
  
 
*Median is <math>6</math>: Then <math>n \le 6</math> and <math>\frac{3+6+9+10+n}{5} = 6 \Longrightarrow n = 2</math>.
 
*Median is <math>6</math>: Then <math>n \le 6</math> and <math>\frac{3+6+9+10+n}{5} = 6 \Longrightarrow n = 2</math>.
 +
 
*Median is <math>9</math>: Then <math>n \ge 9</math> and <math>\frac{3+6+9+10+n}{5} = 9 \Longrightarrow n = 17</math>.  
 
*Median is <math>9</math>: Then <math>n \ge 9</math> and <math>\frac{3+6+9+10+n}{5} = 9 \Longrightarrow n = 17</math>.  
 +
 
*Median is <math>n</math>: Then <math>6 < n < 9</math> and <math>\frac{3+6+9+10+n}{5} = n \Longrightarrow n = 7</math>.
 
*Median is <math>n</math>: Then <math>6 < n < 9</math> and <math>\frac{3+6+9+10+n}{5} = n \Longrightarrow n = 7</math>.
  

Latest revision as of 14:52, 7 August 2017

Problems

The set $\{3,6,9,10\}$ is augmented by a fifth element $n$, not equal to any of the other four. The median of the resulting set is equal to its mean. What is the sum of all possible values of $n$?

$\mathrm{(A)}\ 7\qquad \mathrm{(B)}\ 9\qquad \mathrm{(C)}\ 19\qquad \mathrm{(D)}\ 24\qquad \mathrm{(E)}\ 26$

Solution

The median must either be $6, 9,$ or $n$. Casework:

  • Median is $6$: Then $n \le 6$ and $\frac{3+6+9+10+n}{5} = 6 \Longrightarrow n = 2$.
  • Median is $9$: Then $n \ge 9$ and $\frac{3+6+9+10+n}{5} = 9 \Longrightarrow n = 17$.
  • Median is $n$: Then $6 < n < 9$ and $\frac{3+6+9+10+n}{5} = n \Longrightarrow n = 7$.

All three cases are valid, so our solution is $2 + 7 + 17 = 26 \Longrightarrow \mathrm{(E)}$.

See also

2007 AMC 12A (ProblemsAnswer KeyResources)
Preceded by
Problem 14
Followed by
Problem 16
1 2 3 4 5 6 7 8 9 10 11 12 13 14 15 16 17 18 19 20 21 22 23 24 25
All AMC 12 Problems and Solutions

The problems on this page are copyrighted by the Mathematical Association of America's American Mathematics Competitions. AMC logo.png